LSAT and Law School Admissions Forum

Get expert LSAT preparation and law school admissions advice from PowerScore Test Preparation.

 Administrator
PowerScore Staff
  • PowerScore Staff
  • Posts: 8919
  • Joined: Feb 02, 2011
|
#80578
Complete Question Explanation

Main Point. The correct answer choice is (C).

Answer choice (A):

Answer choice (B):

Answer choice (C): This is the correct answer choice.

Answer choice (D):

Answer choice (E):

This explanation is still in progress. Please post any questions below!
User avatar
 jcg16e
  • Posts: 2
  • Joined: May 31, 2021
|
#87473
Hello, could you please explain why the correct answer is C and not D?
User avatar
 christinecwt
  • Posts: 74
  • Joined: May 09, 2022
|
#95556
Hi Team - can anyone explain why the Answer Choice is C not E? Thanks!
User avatar
 katehos
PowerScore Staff
  • PowerScore Staff
  • Posts: 184
  • Joined: Mar 31, 2022
|
#95597
Hi Christine,

Let's start by breaking down the stimulus into premises and conclusions. The first sentence is a premise stating that waters off the Pacific coast have warmed by about 4 degrees in the past 15 years. The second sentence introduces a claim made by some scientists (which we can guess will be countered by the climatologist); the claim by some scientists is that human-generated air pollution caused global warming, which in turn caused the aforementioned water temperature increase. The third sentence introduces both a conclusion and a premise, with the conclusion being that the scientists' claim is far from justified, and the premise being that there are natural cycles of temperature change.

With this in mind, we can prephrase the climatologist's main conclusion: the claim that the temperature increase is due to global warming is not justified.

Turning to answer choice (E), we can see that this does not match our prephrase and strays away from it. Answer choice (E) introduces a conditional relationship between warm waters being caused by temperature changes and warm waters not being caused by global warming. While each of these ideas individually are present in the stimulus, this specific form of logic is not and the answer choice as a whole does not address the climatologist's main conclusion. The climatologist does not argue that if warming was caused by natural changes then warming was not caused by pollution, rather, the climatologist argues that claiming the warming was caused by pollution is not fully justified because natural temperature changes happen. So, we can eliminate (E) because it is not the most accurate way of expressing the climatologist's argument.

Answer choice (C) summarizes the main conclusion by saying that claiming warming is a symptom of pollution is not justified - almost exactly like our prephrase! This answer perfectly encapsulates the conclusion itself: that the scientists' claim is far from justified (which is the beginning of sentence 3).

When answering these questions, it is helpful to keep in mind what the conclusion itself actually is, since your best answer choice will likely be an almost synonymous rephrasing of the conclusion. A lot of the answers on this question are appealing, but remembering that the author is not saying the scientists' claim is definitively wrong, just that it is not fully justified (and offers a counter-explanation as proof).

I hope this helps! :)
-Kate
User avatar
 bebeg3168
  • Posts: 22
  • Joined: Aug 01, 2022
|
#98002
Hi,
I was curious about D. I was stuck between C and D. C said exactly what was in the passage and used the appropriate language. The problem I have is I have often gotten this question type wrong because it was regurgitating the stimulus. I thought D concluded the stimulus good, even thought it was less wordy. Any help?
 Rachael Wilkenfeld
PowerScore Staff
  • PowerScore Staff
  • Posts: 1358
  • Joined: Dec 15, 2011
|
#98452
The problem with answer choice (D) isn't that it restates the stimulus too directly here, bebeg3168, it's that it's restating the wrong part of the stimulus. In a main point question, we are looking for something that is a direct idea from the stimulus which is also the main point. Answer choice (D) supports the main point of the argument. It's because of the fact that the warming may be a natural cycle that man-made global warming is not fully justified. If you look at answer choices (C) and (D) next to each other, you'll see that answer choice (D) supports answer choice (C). In a main point question, the correct answer will not support any other part of the stimulus. It will only be supported by the rest of the argument.

Hope that helps!
User avatar
 TootyFrooty
  • Posts: 74
  • Joined: Oct 13, 2023
|
#104821
can you please share why b is wrong?

And also what Is the exact meaning of "far from justified" I take that as a very strong affirmative and definitive NO but elsewhere I'm seeing people say its not a definitive no.
 Robert Carroll
PowerScore Staff
  • PowerScore Staff
  • Posts: 1783
  • Joined: Dec 06, 2013
|
#105399
TootyFrooty,

The simplest reason that answer choice (B) is wrong is that the author's conclusion is that the conclusion of the scientists is "far from justified". Answer choice (C) says that, and answer choice (B) doesn't. It's a Main Point question, so we want the author's conclusion.

Going further, the author is not saying the scientists' conclusion is false. To say something is "far from justified" means it's just not proven. Take, for instance, Fermat's Last Theorem. This is a mathematical theorem that some guy named Fermat came up with a few centuries ago. The problem was, he had no proof of it! He claimed his proof was too big for him to fit on paper, so he omitted that proof. Since there was no proof of this theorem known at the time Fermat wrote his paper, we could say Fermat's Last Theorem was "far from justified". That doesn't mean it was false! That doesn't mean no justification was possible. It just means that no justification had in fact been given. "Far from X" does mean something like "X isn't the case", but note that the author says something is far from justified, not far from true. So the author is denying the justification, not the truth, of the scientists' conclusion. I think it's a distraction to get into whether "far from X" means X could be the case - that's not the problem. The problem is that "far from justified" isn't the same at all as "far from true."

Robert Carroll

Get the most out of your LSAT Prep Plus subscription.

Analyze and track your performance with our Testing and Analytics Package.